Identificarse Registrarse

Psu
Enseñanza Básica
Enseñanza Media
Universidad
Olimpiadas
Comunidad



2 Páginas: V   1 2 >  
Reply to this topicStart new topic
> APMO 2007, Ssp: 1,3
Luffy
mensaje Mar 29 2007, 05:05 PM
Publicado: #1


Dios Matemático Supremo
Ícono de Grupo

Grupo: Usuario FMAT
Mensajes: 556
Registrado: 16-August 06
Desde: Rio de Janeiro
Miembro Nº: 1.950
Nacionalidad:
Colegio/Liceo: Instituto Nacional
Universidad: Instituto Nacional de Matematica Pura e Aplicada (IMPA)
Sexo:



TEX: \noindent \underline{$Problema\ 1$} Sea $\mathcal{S}$ un conjunto de 9 n\'umeros enteros distintos con la siguiente propiedad. Para cada $a\in\mathcal{S}$ si $p$ es factor primo de $a$ entonces $p\le 3$. Puruebe que $\mathcal{S}$ contiene tres enteros distintos $r_1$, $r_2$, $r_3$ tal que $r_1\cdot r_2\cdot r_3=n^3$ para alg\'un $n\in\mathbb{Z}$.

Solucion: (Pendiente)

TEX: \noindent \underline{$Problema\ 2$} Sea $ABC$ un tri\'angulo con \'angulos agudos tal que $\angle BAC=60^o$ y $AB>BC$. Sea $I$ el incentro y $H$ el ortocentro del tri\'angulo $ABC$. Pruebe que $2\angle AHI=3\angle ABC$.

Solucion:


TEX: \noindent \underline{$Problema\ 3$} Considere $n$ discos en el plano $C_1,C_2,...,C_n$ con la propiedad de que para cada $1\le i<n$ el centro de $C_i$ est\'a en la circunferencia de $C_{i+1}$ y el centro de $C_n$ est\'a en la circunferencia de $C_1$. Define el puntaje de un arreglo de $n$ discos, como descrito, como el n\'umero de pares $(i,j)$ para los cuales $C_i$ contiene propiamente a $C_j$. Determine el m\'aximo puntaje posible de un arreglo de $n$-discos.

Solucion: (Pendiente)

TEX: \noindent \underline{$Problema\ 4$} Sean $x$, $y$, $z$ n\'umeros reales positivos tales que $\sqrt{x}+\sqrt{y}+\sqrt{z}=1$. Pruebe que:\\<br />\begin{center}<br />$\dfrac{x^2+yz}{\sqrt{2x^2(y+z)}}+\dfrac{y^2+zx}{\sqrt{2y^2(z+x)}}+\dfrac{z^2+xy}{\sqrt{2z^2(x+y)}}\ge 1$<br />\end{center}<br />

Solucion:


TEX: \noindent \underline{$Problema\ 5$} Un arreglo de $5\times 5$ est\'a defectuoso, de tal forma que al cambiar el estado de la luz situada en el lugar $(i,j)$ entonces tambi\'en cambian su estado las luces adyacentes en la fila $i$ y la columna $j$. Inicialmente todas las luces est\'an apagadas y luego de un cierto n\'umero de movimientos exactamente una luz queda encendida. Encuentre todas las posibles posiciones de esta luz.

Solucion:
Go to the top of the page
 
+Quote Post
Luffy
mensaje Mar 29 2007, 05:26 PM
Publicado: #2


Dios Matemático Supremo
Ícono de Grupo

Grupo: Usuario FMAT
Mensajes: 556
Registrado: 16-August 06
Desde: Rio de Janeiro
Miembro Nº: 1.950
Nacionalidad:
Colegio/Liceo: Instituto Nacional
Universidad: Instituto Nacional de Matematica Pura e Aplicada (IMPA)
Sexo:



Solución P4:

TEX:  Asumamos sin perdidad de generalidad que $x\ge y\ge z>0$ . Luego debe cumplirse que:

TEX:  $xy(x-y)+z(x^2-y^2)\ge 0 \wedge xz(x-z)+y(x^2-z^2)\ge 0$

TEX:  $x^2(y+z)\ge y^2(z+x) \wedge x^2(y+z)\ge z^2(x+y)$

TEX:  $\dfrac{(y-x)(y-z)}{\sqrt{y^2(z+x)}}\ge \dfrac{(y-x)(y-z)}{\sqrt{x^2(y+z)}} \wedge \dfrac{(z-x)(z-y)}{\sqrt{z^2(x+y)}}\ge \dfrac{(z-x)(z-y)}{\sqrt{x^2(y+z)}}$

TEX:  Ahora bien, sabiendo que (se desprende de la aplicaci\'on de $MA\ge MG$):

TEX:  $x^2+y^2+z^2\ge  xy+yz+zx$

TEX:  $x^2+yz-x(y+z)+y^2+zx-y(z+x)+z^2+xy-z(x+y)\ge 0$

TEX:  $(x-y)(x-z)+(y-x)(y-z)+(z-x)(z-y)\ge 0$

TEX:  $\dfrac{(x-y)(x-z)}{\sqrt{x^2(y+z)}}+\dfrac{(y-x)(y-z)}{\sqrt{x^2(y+z)}}+\dfrac{(z-x)(z-y)}{\sqrt{x^2(y+z)}}\ge 0$

TEX:  Entonces por transitividad se obtiene:

TEX:  $\dfrac{(x-y)(x-z)}{\sqrt{x^2(y+z)}}+\dfrac{(y-x)(y-z)}{\sqrt{y^2(z+x)}}+\dfrac{(z-x)(z-y)}{\sqrt{z^2(x+y)}}\ge 0$

TEX:  $\dfrac{(x-y)(x-z)}{\sqrt{2x^2(y+z)}}+\dfrac{(y-x)(y-z)}{\sqrt{2y^2(z+x)}}+\dfrac{(z-x)(z-y)}{\sqrt{2z^2(x+y)}}\ge 0$

TEX:  $\dfrac{x^2+yz-x(y+z)}{\sqrt{2x^2(y+z)}}+\dfrac{y^2+zx-y(z+x)}{\sqrt{2y^2(z+x)}}+\dfrac{z^2+xy-z(x+y)}{\sqrt{2z^2(x+y)}}\ge 0$

TEX:  $\dfrac{x^2+yz}{\sqrt{2x^2(y+z)}}-\sqrt{\dfrac{y+z}{2}}+\dfrac{y^2+zx}{\sqrt{2y^2(z+x)}}-\sqrt{\dfrac{z+x}{2}}+\dfrac{z^2+xy}{\sqrt{2z^2(x+y)}}-\sqrt{\dfrac{x+y}{2}}\ge 0$

TEX:  $\dfrac{x^2+yz}{\sqrt{2x^2(y+z)}}+\dfrac{y^2+zx}{\sqrt{2y^2(z+x)}}+\dfrac{z^2+xy}{\sqrt{2z^2(x+y)}}\ge \sqrt{\dfrac{x+y}{2}}+\sqrt{\dfrac{y+z}{2}}+\sqrt{\dfrac{z+x}{2}}$

TEX:  Pero por $MC\ge MA$:

TEX:  $\sqrt{\dfrac{x+y}{2}}+\sqrt{\dfrac{y+z}{2}}+\sqrt{\dfrac{z+x}{2}}\ge \dfrac{\sqrt{x}+\sqrt{y}}{2}+\dfrac{\sqrt{y}+\sqrt{z}}{2}+\dfrac{\sqrt{z}+\sqrt{x}}{2}$

TEX:  $\sqrt{\dfrac{x+y}{2}}+\sqrt{\dfrac{y+z}{2}}+\sqrt{\dfrac{z+x}{2}}\ge \dfrac{2(\sqrt{x}+\sqrt{y}+\sqrt{z})}{2}=1$

TEX:  Finalmente por transitividad:

TEX:  $\boxed{\dfrac{x^2+yz}{\sqrt{2x^2(y+z)}}+\dfrac{y^2+zx}{\sqrt{2y^2(z+x)}}+\dfrac{z^2+xy}{\sqrt{2z^2(x+y)}}\ge 1}$

TEX:  Ocurriendo la igualdad para $\boxed{x=y=z=\dfrac{1}{9}}$

carita2.gif carita2.gif
Go to the top of the page
 
+Quote Post
Vicho_Correa
mensaje Jun 4 2007, 11:13 PM
Publicado: #3


Matemático
Ícono de Grupo

Grupo: Usuario FMAT
Mensajes: 40
Registrado: 19-July 06
Desde: Conce, Jazz Capitol
Miembro Nº: 1.716
Nacionalidad:
Sexo:



Solución P2:


TEX: <br />\[<br />\begin{gathered}<br />  {\text{Tengamos en cuenta que H debe estar al interior del }}\vartriangle ABC.{\text{ Sea D el pie de la altura bajada }} \hfill \\<br />  {\text{desde C}}{\text{. Notemos que I no puede estar al interior del }}\vartriangle ADC.{\text{ En efecto}}{\text{, de no ser as\'i }}{\text{, se }} \hfill \\<br />  {\text{tendr\'ia que }}\angle ACI < 30 \Rightarrow \angle ACB < 60.{\text{ Adem\'as}}{\text{, I no puede estar a interior del }}\vartriangle BHC.{\text{ En }} \hfill \\<br />  {\text{efecto}}{\text{, de no ser as\'i }}{\text{, se tendr\'ia que }}\angle ABI > 30 \Rightarrow \angle ABC > 60.{\text{ Luego}}{\text{, I debe estar al interior }} \hfill \\<br />  {\text{del }}\vartriangle DBH.{\text{ Sea }}\angle {\text{ACB = }}\beta {\text{. Notemos que }}\angle {\text{BHC = 120 y que }}\angle {\text{BIC = 180 - }}\left( {\frac{\beta }<br />{{\text{2}}} + 60 - \frac{\beta }<br />{{\text{2}}}} \right) = .{\text{ }} \hfill \\<br />  {\text{120. Luego}}{\text{, el cuadril\'atero BIHC es c\'iclico}}{\text{, de donde }}\angle {\text{IHB = }}\frac{\beta }<br />{{\text{2}}}.{\text{ Adem\'as}}{\text{, }}\angle AHB = 180 - \beta {\text{, de }} \hfill \\<br />  {\text{donde }}\angle AHI = 180 - \frac{{3\beta }}<br />{2}{\text{. Ahora es f\'acil concluir lo pedido}}{\text{, teniendo en cuenta que }} \hfill \\<br />  \angle ABC = 120 - \beta . \hfill \\ <br />\end{gathered} <br />\]<br />



Go to the top of the page
 
+Quote Post
S. E. Puelma Moy...
mensaje Jun 15 2007, 02:10 PM
Publicado: #4


Dios Matemático Supremo
Ícono de Grupo

Grupo: Administrador
Mensajes: 2.706
Registrado: 13-May 05
Desde: Santiago de Chile
Miembro Nº: 10
Nacionalidad:
Colegio/Liceo: Instituto Nacional
Sexo:



Resultó simple la solución al siguiente problema. Si quieren más detalles, pueden preguntar.

Solución al problema 3

Dados dos números: i, j en el conjunto {1,2,...,n}, diremos que el par (i,j) es bueno si el círculo Ci contiene propiamente al círculo Cj. Algunas propiedades evidentes (considerando i, j como en esta definición):
  • (i,i) no es un par bueno
  • Si i es distinto de j, entonces los pares (i,j) y (j,i) no son buenos, simultáneamente
Por lo tanto, el puntaje de un arreglo (i.e.: el número total de pares buenos) es menor o igual que TEX: $\dfrac{n(n-1)}2$. Esta cota superior puede ser alcanzada, como muestra el siguiente arreglo (daré una descripción cartesiana de los círculos):
  • Para cada número i en el conjunto {1,2,...,n-1}: El centro de Ci tiene coordenadas (2i-1,0); el radio de Ci mide 2i-1
  • En centro de Cn tiene coordenadas (0,0); el radio de Cn mide 2n
Ahora, sólo queda explicar todos los detalles: que este arreglo cumple las hipótesis del problema, y que su puntaje es TEX: $\dfrac{n(n-1)}2$.


--------------------
Sebastián Elías Puelma Moya
Administrador FMAT
Go to the top of the page
 
+Quote Post
Vicho_Correa
mensaje Jul 25 2007, 12:26 AM
Publicado: #5


Matemático
Ícono de Grupo

Grupo: Usuario FMAT
Mensajes: 40
Registrado: 19-July 06
Desde: Conce, Jazz Capitol
Miembro Nº: 1.716
Nacionalidad:
Sexo:



CITA(xsebastian @ Jun 15 2007, 03:10 PM) *
Para cada i en el conjunto {1,2,...,n-1}, la circunferencia de Ci pasa por el punto de coordenadas (2i,0), que es el centro del círculo Ci+1.


Pero de acuerdo al enunciado se supone que es la circunferencia TEX: $C_{i+1}$ la que debe pasar por el centro de TEX: $C_{i}$. dunno.gif

Mensaje modificado por Vicho_Correa el Jul 25 2007, 12:28 AM
Go to the top of the page
 
+Quote Post
Luffy
mensaje Aug 5 2007, 01:06 AM
Publicado: #6


Dios Matemático Supremo
Ícono de Grupo

Grupo: Usuario FMAT
Mensajes: 556
Registrado: 16-August 06
Desde: Rio de Janeiro
Miembro Nº: 1.950
Nacionalidad:
Colegio/Liceo: Instituto Nacional
Universidad: Instituto Nacional de Matematica Pura e Aplicada (IMPA)
Sexo:



Ups es cierto, xsebastian tiene que arreglar ese problema, por mientras el problema sigue en pie.

Gracias
Go to the top of the page
 
+Quote Post
caf_tito
mensaje Aug 19 2007, 05:30 PM
Publicado: #7


Dios Matemático Supremo
Ícono de Grupo

Grupo: Colaborador Silver
Mensajes: 1.605
Registrado: 25-June 05
Miembro Nº: 123
Colegio/Liceo: Liceo Oscar Castro Rancagua
Universidad: Universidad de Chile
Sexo:



Ojalá que esté bien, además mi redacción no es de las mejores.

TEX: \[<br />\begin{gathered}<br />  \boxed{S_1 } \hfill \\<br />  r_1  = 2^{q_1 } 3^{p_1 }  \hfill \\<br />  r_2  = 2^{q_2 } 3^{p_2 }  \hfill \\<br />  r_3  = 2^{q_3 } 3^{p_3 }  \hfill \\<br />   \vdots  \hfill \\<br />  r_9  = 2^{q_9 } 3^{p_9 }  \hfill \\<br />  {\text{Se cumple lo pedido}} \Leftrightarrow \left. 3 \right|q_x  + q_y  + q_z  \wedge \left. 3 \right|p_x  + p_y  + p_z  \hfill \\<br />   \hfill \\<br />  {\text{Como todo numero divido por 3}}{\text{, deja resto 0}} \vee {\text{1}} \vee 2,{\text{ los elementos de }}S \hfill \\<br />  {\text{podemos clasificarlos en estos 9 subconjuntos}} \hfill \\<br />  2^{q_a } 3^{p_a }  \in S_{0a}  \Leftrightarrow \forall q_a ,p_a {\text{ se cumple que: }}q_a  \equiv 0\bmod 3 \wedge p_a  \equiv 0\bmod 3 \hfill \\<br />  2^{q_b } 3^{p_b }  \in S_{0b}  \Leftrightarrow \forall q_b ,p_b {\text{ se cumple que: }}q_b  \equiv 0\bmod 3 \wedge p_b  \equiv 1\bmod 3 \hfill \\<br />  2^{q_c } 3^{p_c }  \in S_{0c}  \Leftrightarrow \forall q_c ,p_c {\text{ se cumple que: }}q_c  \equiv 0\bmod 3 \wedge p_c  \equiv 2\bmod 3 \hfill \\<br />  2^{q_d } 3^{p_d }  \in S_{1a}  \Leftrightarrow \forall q_d ,p_d {\text{ se cumple que: }}q_d  \equiv 1\bmod 3 \wedge p_d  \equiv 0\bmod 3 \hfill \\<br />  2^{q_e } 3^{p_e }  \in S_{1b}  \Leftrightarrow \forall q_e ,p_e {\text{ se cumple que: }}q_e  \equiv 1\bmod 3 \wedge p_e  \equiv 1\bmod 3 \hfill \\<br />  2^{q_f } 3^{p_f }  \in S_{1c}  \Leftrightarrow \forall q_f ,p_f {\text{ se cumple que: }}q_f  \equiv 1\bmod 3 \wedge p_f  \equiv 2\bmod 3 \hfill \\<br />  2^{q_g } 3^{p_g }  \in S_{2a}  \Leftrightarrow \forall q_g ,p_g {\text{ se cumple que: }}q_g  \equiv 2\bmod 3 \wedge p_g  \equiv 0\bmod 3 \hfill \\<br />  2^{q_h } 3^{p_h }  \in S_{2b}  \Leftrightarrow \forall q_h ,p_h {\text{ se cumple que: }}q_h  \equiv 2\bmod 3 \wedge p_h  \equiv 1\bmod 3 \hfill \\<br />  2^{q_i } 3^{p_i }  \in S_{2c}  \Leftrightarrow \forall q_i ,p_i {\text{ se cumple que: }}q_i  \equiv 2\bmod 3 \wedge p_i  \equiv 2\bmod 3 \hfill \\ <br />\end{gathered} <br />\]

TEX: \[<br />\begin{gathered}<br />  {\text{Caso 1}} \hfill \\<br />  {\text{Si  al menos 3 de los elementos pertenecen al mismo subconjunto }}S_i {\text{ se cumple}} \hfill \\<br />  {\text{lo pedido}}{\text{.}} \hfill \\<br />  {\text{Dem 1:}} \hfill \\<br />  {\text{Al sumar tres numeros con el mismo resto al ser divido por 3}}{\text{, se obtiene que}} \hfill \\<br />  {\text{la suma es divisible por 3}} \hfill \\<br />  a \equiv 0\bmod 3,b \equiv 0\bmod 3,c \equiv 0\bmod 3 \Rightarrow a + b + c \equiv 0\bmod 3 \hfill \\<br />  a \equiv 1\bmod 3,b \equiv 1\bmod 3,c \equiv 1\bmod 3 \Rightarrow a + b + c \equiv 3 \equiv 0\bmod 3 \hfill \\<br />  a \equiv 2\bmod 3,b \equiv 2\bmod 3,c \equiv 2\bmod 3 \Rightarrow a + b + c \equiv 6 \equiv 0\bmod 3 \hfill \\ <br />\end{gathered} <br />\]


TEX: \[<br />\begin{gathered}<br />  {\text{Caso 2}} \hfill \\<br />  {\text{Supongamos que a lo mas hay elementos por cada subconjunto }}\left( {{\text{para que no}}} \right. \hfill \\<br />  \left. {{\text{se cumpla el Caso 1}}} \right). \hfill \\<br />  {\text{Cada subconjunto posee a lo mas dos elementos}}{\text{, y como son 9 elementos}} \hfill \\<br />  {\text{habran por lo menos 5 subconjuntos con elementos}}{\text{. Notemos que podemos}} \hfill \\<br />  {\text{clasificar a estos subconjuntos en las siguientes categorias:}} \hfill \\<br />  \boxed{S_{0a} ,S_{0b} ,S_{0c} } \hfill \\<br />  \boxed{S_{1a} ,S_{1b} ,S_{1c} } \hfill \\<br />  \boxed{S_{2a} ,S_{2b} ,S_{2c} } \hfill \\<br />   \hfill \\<br />  \boxed{S_{0a} ,S_{1a} ,S_{2a} } \hfill \\<br />  \boxed{S_{0b} ,S_{1b} ,S_{2b} } \hfill \\<br />  \boxed{S_{0c} ,S_{1c} ,S_{2c} } \hfill \\<br />   \hfill \\<br />  \boxed{S_{0a} ,S_{1b} ,S_{2c} } \hfill \\<br />  \boxed{S_{0a} ,S_{1c} ,S_{2b} } \hfill \\<br />  \boxed{S_{0b} ,S_{1a} ,S_{2c} } \hfill \\<br />  \boxed{S_{0b} ,S_{1c} ,S_{2a} } \hfill \\<br />  \boxed{S_{0c} ,S_{1a} ,S_{2b} } \hfill \\<br />  \boxed{S_{0c} ,S_{1b} ,S_{2a} } \hfill \\<br />   \hfill \\<br />  {\text{Es evidente que siempre habra una categoria donde todos sus subconjuntos}} \hfill \\<br />  {\text{tengan por lo menos un elemento}}{\text{.}} \hfill \\<br />   \hfill \\<br />  {\text{Finalmente tendremos que demostrar; si cada subconjunto de una categoria}} \hfill \\<br />  {\text{posee al menos un elemento se cumple lo pedido}}{\text{.}} \hfill \\ <br />\end{gathered} <br />\]


TEX: \[<br />\begin{gathered}<br />  Lema \hfill \\<br />  {\text{Las categorias de la forma }}\boxed{S_{ta} ,S_{tb} ,S_{tc} }{\text{ cumple lo pedido}} \hfill \\<br />  {\text{Dem 2:}} \hfill \\<br />  {\text{Notemos que todos los conjuntos de la forma }}S_{ta} ,S_{tb} ,S_{tc} {\text{ posee cada uno}} \hfill \\<br />  {\text{un }}q{\text{ con el mismo resto}}{\text{, entonces por la Dem }}1{\text{ se cumple lo pedido}} \hfill \\<br />  {\text{para }}q. \hfill \\<br />  {\text{Notemos tambien que }} \hfill \\<br />  S_{ta} {\text{ posee un }}p_{ta}  \equiv 0\bmod 3 \hfill \\<br />  S_{tb} {\text{ posee un }}p_{tb}  \equiv 1\bmod 3 \hfill \\<br />  S_{tc} {\text{ posee un }}p_{tc}  \equiv 2\bmod 3 \hfill \\<br />   \Rightarrow {\text{ }}p_{ta}  + p_{tb}  + p_{tc}  \equiv 3 \equiv 0\bmod 3 \hfill \\<br />  {\text{Demostrando asi nuestro }}Lema \hfill \\<br />   \hfill \\<br />  Lema{\text{ 2}} \hfill \\<br />  {\text{Las categorias de la forma }}\boxed{S_{0p} ,S_{1p} ,S_{2p} }{\text{ cumple lo pedido}} \hfill \\<br />  Dem{\text{ 3:}} \hfill \\<br />  {\text{Basta leer la dem 2 y cambiar las }}q{\text{ por las }}p. \hfill \\<br />\end{gathered} <br />\]

TEX: \[<br />\begin{gathered}<br />  {\text{Lema 3}} \hfill \\<br />  {\text{Las categorias de la forma }}S_{0m} ,S_{1n} ,S_{2r} {\text{ cumplen lo pedido}}{\text{.}} \hfill \\<br />  {\text{Dem 4}} \hfill \\<br />  {\text{Como definimos anteriormente es claro que }}S_{0m} {\text{ posee un 2}}^{q_m } 3^{p_m } {\text{ tal que}} \hfill \\<br />  q_m  \equiv 0\bmod 3.{\text{ Analogamente }}S_{1n} {\text{ y }}S_{2r} {\text{ poseen }}q's{\text{ tal que son }}q_n  \equiv 1\bmod 3 \hfill \\<br />  {\text{y }}q_r  \equiv 2\bmod 3. \hfill \\<br />  {\text{Luego notemos que }}S_{0m} ,S_{1n} ,S_{2r} {\text{ poseen cada uno un 2}}^q 3^p {\text{ tal que sus}} \hfill \\<br />  p{\text{ tienen residuos distintos entre si al ser dividos por tres}}{\text{.}} \hfill \\<br />  {\text{Tomando la demo 2 el razonamiento es analogo para }}q{\text{ y }}p \hfill \\<br />   \hfill \\<br />  {\text{Con esto finaliza la demostracion pues siempre se cumple el Caso 1 o el Caso 2}} \hfill \\<br />  {\text{P}}{\text{.S: Notar que el }}Lema{\text{ }}3{\text{ es mas general que el }}Lema{\text{ }}2. \hfill \\ <br />\end{gathered} <br />\]

Editado.

Mensaje modificado por caf_tito el Aug 21 2007, 10:21 PM


--------------------
Go to the top of the page
 
+Quote Post
Luffy
mensaje Aug 19 2007, 07:23 PM
Publicado: #8


Dios Matemático Supremo
Ícono de Grupo

Grupo: Usuario FMAT
Mensajes: 556
Registrado: 16-August 06
Desde: Rio de Janeiro
Miembro Nº: 1.950
Nacionalidad:
Colegio/Liceo: Instituto Nacional
Universidad: Instituto Nacional de Matematica Pura e Aplicada (IMPA)
Sexo:



CITA(caf_tito @ Aug 19 2007, 06:30 PM) *
TEX: \[<br />\begin{gathered}<br />  {\text{Caso 2}} \hfill \\<br />  {\text{Supongamos que a lo mas hay 2 elementos por cada subconjunto (para que no}} \hfill \\<br />  {\text{se cumpla el Caso 1)}}{\text{.}} \hfill \\<br />  {\text{Cada subconjunto posee a lo mas dos elementos}}{\text{,y como son 9 elementos}} \hfill \\<br />  {\text{habran por lo menos 5 subconjuntos con elementos}}{\text{.Notemos que podemos}} \hfill \\<br />  {\text{clasificar a estos subconjuntos en las siguientes categorias:}} \hfill \\<br />  \boxed{S_{0a} ,S_{0b} ,S_{0c} } \hfill \\<br />  \boxed{S_{1a} ,S_{1b} ,S_{1c} } \hfill \\<br />  \boxed{S_{2a} ,S_{2b} ,S_{2c} } \hfill \\<br />  \boxed{S_{0a} ,S_{1a} ,S_{2a} } \hfill \\<br />  \boxed{S_{0b} ,S_{1b} ,S_{2b} } \hfill \\<br />  \boxed{S_{0c} ,S_{1c} ,S_{2c} } \hfill \\<br />  {\text{Es evidente que siempre habra una categoria donde todos sus}} \hfill \\<br />  {\text{subconjuntos tengan por lo menos un elemento}}{\text{.}} \hfill \\<br />   \hfill \\<br />  {\text{Finalemente tendremos que demostrar que si cada subconjunto de una categoria}} \hfill \\<br />  {\text{posee al menos un elemento se cumple lo pedido}}{\text{.}} \hfill \\ <br />\end{gathered} <br />\]


Por qué es evidente??? Contraejemplo: TEX: $S_{0a} ,S_{0b},S_{1a} ,S_{1c},S_{2b}$

Creo que tu idea de agrupar es muy buena, pero creo que te faltó poner talvez unas 6 categorias más que debes analizar para dar por correcta tu solución whistling.gif

Saludos jpt_chileno.gif
Go to the top of the page
 
+Quote Post
caf_tito
mensaje Aug 19 2007, 07:54 PM
Publicado: #9


Dios Matemático Supremo
Ícono de Grupo

Grupo: Colaborador Silver
Mensajes: 1.605
Registrado: 25-June 05
Miembro Nº: 123
Colegio/Liceo: Liceo Oscar Castro Rancagua
Universidad: Universidad de Chile
Sexo:



CITA(Luffy @ Aug 19 2007, 08:23 PM) *
Por qué es evidente??? Contraejemplo: TEX: $S_{0a} ,S_{0b},S_{1a} ,S_{1c},S_{2b}$

Creo que tu idea de agrupar es muy buena, pero creo que te faltó poner talvez unas 6 categorias más que debes analizar para dar por correcta tu solución whistling.gif

Saludos jpt_chileno.gif


mamon.gif tienes razón jpt_blush.gif .


--------------------
Go to the top of the page
 
+Quote Post
Luffy
mensaje Aug 25 2007, 07:24 PM
Publicado: #10


Dios Matemático Supremo
Ícono de Grupo

Grupo: Usuario FMAT
Mensajes: 556
Registrado: 16-August 06
Desde: Rio de Janeiro
Miembro Nº: 1.950
Nacionalidad:
Colegio/Liceo: Instituto Nacional
Universidad: Instituto Nacional de Matematica Pura e Aplicada (IMPA)
Sexo:



Ahora que agregaste estas 6 nuevas categorias, es cierto que siempre hay una con los 3 subconjuntos con almenos 1 elemento; pero no es tan evidente como tu lo plateas, me gustaria que argumentaras bien eso para darla por correcta de modo que quede a la comprensión de todos.

Saludos
Go to the top of the page
 
+Quote Post

2 Páginas: V   1 2 >
Reply to this topicStart new topic
1 usuario(s) está(n) leyendo esta discusión (1 invitado(s) y 0 usuario(s) anónimo(s))
0 miembro(s):

 

Versión Lo-Fi Fecha y Hora actual: 23rd November 2024 - 06:49 PM